Ginger's baby weighed only 3 3/8 pounds when it was born. The doctor said it could not leave the hospital until it weighed 5 1/2 pounds. How many pounds must the baby gain before it can go home to Ginger ?
Solve in fraction form

Answers

Answer 1

Answer:

The baby must gain 2 1/8 more pounds before he/she can go to Ginger's house.Step-by-step explanation:11/2 - 27/8 = Required pounds for the baby to gain=> 44/8 - 27/8 = Required pounds for the baby to gain=> 17/8 = Required pounds for the baby to gain=> 2 1/8 = Required pounds for the baby to gainConclusion:

Hence, the baby must gain 2 1/8 more pounds before he/she can go to Ginger's house.

Hoped this helped.

\(BrainiacUser1357\)


Related Questions

Determine whether the sequence \( \left\{a_{n}\right\} \) converges or diverges. If it converges, find its limit. (1) \( a_{n}=\frac{n !}{n^{n}} \) (2) \( a_{n}=\frac{(\ln n)^{\pi}}{\sqrt{n}} \) ((3) a
n

=
ln(n
2
+1)+1
ln(n+1)

(4) a
n

=n
2
(1−cos
n
1

)

Answers

In mathematics, a sequence is a list of numbers that are ordered in a particular way. Sequences can be finite or infinite, and they can be increasing, decreasing, or neither. In this lesson, we will discuss four sequences and their convergence or divergence.

1. The sequence (an) = n!/nⁿ converges to 1 as n approaches infinity.

2. The sequence (an) = \(\frac{\ln(n)^\pi}{\sqrt{n}}\) diverges.

3. The sequence (an) = ln(n²+1) + 1/ln(n+1) converges to 1.

4. The sequence (an) = n²(1-cos(1/n)) converges to 0.

1. The sequence ( \(\left{a_{n}\right}\)) where ( \(a_{n}=\frac{n !}{n^{n}}\) ) converges to 1.

This can be shown using the Stirling approximation, which states that

\(n! \approx \sqrt{2 \pi n} \left(\frac{n}{e}\right)^n\)

Substituting this into the definition of ( \(a_{n\)} ), we get

\(a_{n} \approx \frac{\sqrt{2 \pi n} \left(\frac{n}{e}\right)^n}{n^n} = \frac{1}{\sqrt{2 \pi}}\)

As n approaches infinity, the value of ( \(a_{n}\) ) approaches 1.

2. The sequence ( \(\left{a_{n}\right}\)) where ( \(a_{n}=\frac{(\ln n)^{\pi}}{\sqrt{n}}\) ) diverges.

This can be shown using the fact that the logarithm function is unbounded, which means that for any positive number k, there exists a natural number n such that ln(n) > k. This means that for any positive number M, there exists a natural number N such that ( \(a_{N}=\frac{(\ln N)^{\pi}}{\sqrt{N}} > M\) ). This shows that the sequence ( \(\left{a_{n}\right}\) ) does not have a limit, and therefore diverges.

3. The sequence ( \(\left{a_{n}\right}\) ) where ( \(a_{n}=\ln(n^2+1)+\frac{1}{\ln(n+1)}\)) converges to 1.

This can be shown using the fact that the logarithm function is continuous and increasing, which means that for any two real numbers x and y, ln(x) < ln(y) if and only if x < y. This means that for any natural number n, the sequence ( \(a_{n}=n^2(1-\cos(1/n))\)) is increasing. Since the sequence is increasing, it must converge to a limit. The limit of the sequence is the value of the sequence at the limit point, which is 1.

4. The sequence ( \(\left{a_{n}\right}\)) where ( \(a_{n}=n^2(1-\cos(1/n))\) ) converges to 0.

This can be shown using the fact that the cosine function oscillates between -1 and 1. This means that for any natural number n, the value of ( \(a_{n}\) ) is between 0 and n². Since the sequence is bounded, it must converge. The limit of the sequence is the value of the sequence at the limit point, which is 0.

To know more about the Sequences refer here,

https://brainly.com/question/32716499#

#SPJ11

used to measure the center of a set of values. good for summarizing values that are generally pretty similar to each other.

Answers

Mean is used to measure the center of a set of values. It is good for summarizing values which are generally pretty similar to each other.

What is mean and its function?

Mean is more usually referred to as the average. It is calculated by adding up all of the values and dividing by the total number of values. It is a good way for summarizing the center of values which are commonly very similar to each other.

The mean is the most often used measure of central tendency since it uses all values in the data set to give us an average. For data from skewed distributions, the median is better than the mean because it is not influenced by large values.

Learn more about mean at: https://brainly.com/question/30090607

#SPJ4

Although part of your question is missing, you might be referring to this full question: _________ is used to measure the center of a set of values. It is good for summarizing values that are generally pretty similar to each other.

Slope=5, goes through the point (2,7) what is the answer

Answers

Answer:

slope-intercept form: y=5x-3 point-slope form: y-7=5*(x-2)

Step-by-step explanation:

Y-y1=m(x-x1)

plug in the values and you get y=5x-3

\(\huge\boxed{\mathcal{HELLO!:)}}\)

Since we are given the slope of the line and a point that it passes through, we can easily determine the equation of the line.

First of all, we need to write it in Point-Slope Form:

\(\huge\boxed{\boxed{\rm{y-y1=m(x-x1)}}}\)

Where y1 is the y-coordinate of the point, m is the slope, and x is the x-coordinate of the point.

Plug in the values and solve:

\(\huge\rm{y-7=5(x-2)\)

\(\rm{y-7=5x-10\)

\(\rm{y=5x-10+7}\\\rm{y=5x-3\)

\(\huge\boxed{\mathbb{ANSWER:{\boxed{\bold{y=5x-3}}}}}\)

\(\bigstar\star\) Hope it helps! Enjoy your day!

\(\rm{FabulousKingdom:)\)

Rectangle A has side lengths of
6
cm
6 cm6, start text, space, c, m, end text and
3.5
cm
3.5 cm3, point, 5, start text, space, c, m, end text. The side lengths of rectangle B are proportional to the side lengths of rectangle A.
What could be the side lengths of rectangle B?
Choose 2 answers:

Answers

The possible side lengths of rectangle B are 12 cm and 7 cm, or 9 cm and 5.25 cm, etc.

What are the side lengths of rectangle B?

The side length of rectangle B is calculated as follows;

Side length of rectangle A = 6 cm and 3.5 cm

If the two rectangles are proportional, the possible side lengths of rectangle B is calculated as follows;

Length of B = 2 x 6 cm = 12 cm

Width of B = 2  x 3.5 cm = 7 cm

or

Length of B = 1.5 x 6 cm = 9 cm

Width of B =1.5  x 3.5 cm = 5.25 cm

Thus, the side lengths of rectangle B will be increasing or decreasing at equal proportion.

Learn more about lengths of rectangles here: https://brainly.com/question/24571594

#SPJ1

What is the range of possible sizes for side x?

What is the range of possible sizes for side x?

Answers

If x is the largest side :

x < 4.1 + 1.3x < 5.4

If 4.1 is the greatest side :

4.1 < x + 1.3x > 2.8

Range of possible sizes for side 'x' :

2.8 < x < 5.4

In which branch of statistics would a researcher acquire twenty-five 2000 Toyota Celicas, drive them until they had a major mechanical failure, record the final mileage, and then write a report for Car and Driver?

Answers

Answer:

The answer is "descriptive statistics".

Step-by-step explanation:

Descriptive statistics are short descriptive factors that add up a production set, that can be either a reflection of a whole or a sample population. It is broken down into cumulative frequency measurements and variance measurements. The study uses descriptive statistics to explain the profile of the respondents. People give simple summaries about the sample as well as the actions.

a bluebird is sitting in her nest 10 feet up in a tree when she looks down and sees a cat and a worm. she decides to use math to determine if the cat is dar enough from the worm to gige hee time to fly down ans get the worm before the cat would read her. the worm is 7 feet from the base of the tree. how far is the cat from the worm?

the distance from the cat to the base of the tree is ___ feet (rounded to the nearest tenth)

the distance from the worm is ____ feet ( rounded to the nearest tenth).

Answers

Answer:

12.2 feet

Step-by-step explanation:

The key to this problem is trigonometry.

Imagine a right triangle. With a height, 10 ft (representing the tree's height), and a base of 7 ft (representing the horizontal distance to the worm).

Now the hypoteneuse length, or otherwise the distance from the worm is unknown.

So this is where we can use the pythagorean theorem.

\(a^2 + b^2 = c^2\)

\(c\) here is the hypoteneuse length, and \(a\) and \(b\) are the other lengths of the triangle.

Since we know a and b we can solve for c.

But first we will square root both sides of the equation to get \(c\) and not  \(c^2\).

\(\sqrt{a^2 + b^2} = c\)

Now just plug in our values for a and b respectively. (It doesn't matter what number replaces a or b, just make sure you are replacing a and b with the 2 known values) Calculate.

\(\sqrt{10^2 + 7^2} = \sqrt{100 + 49} = \sqrt{149} = 12.206....\)

Round to the nearest tenth.

\(12.2\) feet

I NEED HELP ASAP PLEASE AND THANK YOU

Answers

Answer:

help on what???????????

Step-by-step explanation:WHAT IS THIS ??

HOW CAN I HELP U!??

to find where p is increasing, we must decide where p' = 1.7p 1 − p 5600 is positive. assuming that p > 0 always, then we need 1 − p 5600 > 0. therefore, p' is positive whenever

Answers

To find where p is increasing, we must decide where p' = 1.7p(1 − p/5600) is positive. Assuming that p > 0 always, then we need (1 − p/5600) > 0. Therefore, p' is positive whenever p < 5600.

In other words, the function p is increasing whenever the value of p is less than 5600. This is because the derivative p' is positive in this range, meaning that the function p is increasing. As soon as p reaches 5600, the derivative p' becomes negative, meaning that the function p is decreasing.

So, the answer is that p is increasing whenever p < 5600.

Learn more about derivative

brainly.com/question/30365299

#SPJ11

(1.) Convert 25 in base 10 into base 2
(2.)Convert 101011 in base 2 into base 5

Answers

answer here!

sry it is a bit messy

(1.) Convert 25 in base 10 into base 2(2.)Convert 101011 in base 2 into base 5
(1.) Convert 25 in base 10 into base 2(2.)Convert 101011 in base 2 into base 5
(1.) Convert 25 in base 10 into base 2(2.)Convert 101011 in base 2 into base 5

Answer:

1) 11001 base 2

2)133 base 5

1) To convert base 10 into base 2

25/2= 12remainder 1

from this point on, the remainders will be replaced by r

12/2= 6r0

6/2=3r0

3/2=1r1

1/2=0r1

The remainders are then written down from the bottom up

= 11001

2) To convert 101011 base 2 into base 5

101011= 1×2⁵ + 0×2⁴+ 1×2³+ 0×2² + 1×2¹ + 1×2⁰

=1×32 + 0×16 + 1×8+ 0×4+1×2+1×1

=32+0+8+0+2+1

=43 base 10

Then convert 43 base 10 to base 5

43/5=8r3

8/5=1r3

1/5=0r1

=133 base 5

There are 10 marbles in a paper bag, 6 of which are red. The rest are blue. You play a game with your friend. If you draw a red marble, you win. If you draw a blue marble she wins. Is this a fair game?

Answers

No, because you have 1 more marble than her. While you have 6 marbles in the bag, she has 4; there’s a 3/5 chance that you’ll win, and a 2/5 chance that she’ll win.

Seven of the last 25 cars to pass by the
Intersection ersection were SUVS, of the next 50 cars,
How many do you expect to be SUVS?

Answers

Well based on the info given I’d say you would expect to see about 14 more.

Javier drove 3 3/4 miles in 1/3 hour. What was his average speed in miles
10 poin
per hour? *
Your answer

Answers

Answer:

11 1/2 miles per hour is the answer I believe

Find the mean, the median, and the mode(s), if any, for the given data. Round noninteger means to the nearest tenth. (If there ismore than one mode, enter your answer as a comma-separated list. If an answer does not exist, enter DNE.)18,3, 40, 19, 42, 48, 51, 43meanmedianmode(s)

Answers

The data are listed in ascending order to be:

\(3,18,19,40,42,43,48,51\)

MEAN

The mean is the average of a data set. This can be calculated using the formula:

\(Mean=\frac{Sum\text{ }of\text{ }numbers}{Number\text{ }of\text{ }Numbers}\)

From the question, we can calculate the mean to be:

\(\begin{gathered} Mean=\frac{3+18+19+40+42+43+48+51}{8}=\frac{264}{8} \\ Mean=33 \end{gathered}\)

The mean is 33.

MEDIAN

The median is the middle of the set of numbers.

The middle terms are 40 and 42. Therefore, the median will be the average of the two numbers:

\(\Rightarrow\frac{40+42}{2}=41\)

The median is 41.

MODE

The mode is the most common number in a data set.

The modes of the data are 3, 18, 19, 40, 42, 43, 48, and 51.

Solve the given initial value problem.
dx/dt = 6x + y; x(0) = 1
dy/dt = - 4x + y; y(0) = 0

The solution is x(t) = ___ and y(t) = ______ .

Answers

The solutions to the given initial value problem are:

x(t) = c₁\(e^{2\sqrt{2}t }\) + \(e^{-2\sqrt{2}t }\)

y(t) =-\(e^{12it\) + \(e^{-12it\)

Here, we have,

To solve the given initial value problem, we have the following system of differential equations:

dx/dt = 6x + y (1)

dy/dt = -4x + y (2)

Let's solve this system of differential equations step by step:

First, we'll differentiate equation (1) with respect to t:

d²x/dt² = d/dt(6x + y)

= 6(dx/dt) + dy/dt

= 6(6x + y) + (-4x + y)

= 36x + 7y (3)

Now, let's substitute equation (2) into equation (3):

d²x/dt² = 36x + 7y

= 36x + 7(-4x + y)

= 36x - 28x + 7y

= 8x + 7y (4)

We now have a second-order linear homogeneous differential equation for x(t).

Similarly, we can differentiate equation (2) with respect to t:

d²y/dt² = d/dt(-4x + y)

= -4(dx/dt) + dy/dt

= -4(6x + y) + y

= -24x - 3y (5)

Now, let's substitute equation (1) into equation (5):

d²y/dt² = -24x - 3y

= -24(6x + y) - 3y

= -144x - 27y (6)

We have another second-order linear homogeneous differential equation for y(t).

To solve these differential equations, we'll assume solutions of the form x(t) = \(e^{rt}\) and y(t) = \(e^{st}\),

where r and s are constants to be determined.

Substituting these assumed solutions into equations (4) and (6), we get:

r² \(e^{rt}\) = 8 \(e^{rt}\) + 7 \(e^{st}\) (7)

s² \(e^{st}\) = -144 \(e^{rt}\) - 27 \(e^{st}\)(8)

Now, we can equate the exponential terms and solve for r and s:

r² = 8 (from equation (7))

s² = -144 (from equation (8))

Taking the square root of both sides, we get:

r = ±2√2

s = ±12i

Therefore, the solutions for r are r = 2√2 and r = -2√2, and the solutions for s are s = 12i and s = -12i.

Using these solutions, we can write the general solutions for x(t) and y(t) as follows:

x(t) = c₁\(e^{2\sqrt{2}t }\) + c₂\(e^{-2\sqrt{2}t }\) (9)

y(t) = c₃\(e^{12it\) + c₄\(e^{-12it\) (10)

Now, let's apply the initial conditions to find the specific values of the constants c₁, c₂, c₃, and c₄.

Given x(0) = 1, we substitute t = 0 into equation (9):

x(0) = c₁\(e^{2\sqrt{2}(0) }\) + c₂\(e^{-2\sqrt{2}(0) }\)

= c₁ + c₂

= 1

Therefore, c₁ + c₂ = 1. This is our first equation.

Given y(0) = 0, we substitute t = 0 into equation (10):

y(0) = c₃e⁰+ c₄e⁰

= c₃ + c₄

= 0

Therefore, c₃ + c₄ = 0. This is our second equation.

To solve these equations, we can eliminate one of the variables.

Let's solve for c₃ in terms of c₄:

c₃ = -c₄

Substituting this into equation (1), we get:

-c₄ + c₄ = 0

0 = 0

Since the equation is true, c₄ can be any value. We'll choose c₄ = 1 for simplicity.

Using c₄ = 1, we find c₃ = -1.

Now, we can substitute these values of c₃ and c₄ into our equations (9) and (10):

x(t) = c₁\(e^{2\sqrt{2}t }\) + c₂\(e^{-2\sqrt{2}t }\)

= c₁\(e^{2\sqrt{2}t }\) + (1)\(e^{-2\sqrt{2}t }\)

= c₁\(e^{2\sqrt{2}t }\) + \(e^{-2\sqrt{2}t }\)

we have,

y(t) = c₃\(e^{12it\) + c₄\(e^{-12it\)

= (-1)\(e^{12it\) + (1)\(e^{-12it\)

= -\(e^{12it\) + \(e^{-12it\)

Thus, the solutions to the given initial value problem are:

x(t) = c₁\(e^{2\sqrt{2}t }\) + \(e^{-2\sqrt{2}t }\)

y(t) =-\(e^{12it\) + \(e^{-12it\)

For more such questions on Initial-value:

brainly.com/question/2546258

#SPJ4

Gavin deposited $200 into his savings account that is compounded semi-annually at an interest rate of 9%. Gavin hoped he would have enough money in 8 years to buy a $400 gaming console for college. Is Gavin correct? Write and solve an equation, showing your work to justify your answer​

Answers

Answer:

His answer was correct:

(200*0.09)=18

18*16=288

200+288=488

Solve for h.

–2 − 10h = –11h − 20

Answers

。☆✼★ ━━━━━━━━━━━━━━  ☾  

-2 - 10h = -11h - 20

+ 2

- 10h = -11h - 18

+ 11h

h = -18

Have A Nice Day ❤    

Stay Brainly! ヅ    

- Ally ✧    

。☆✼★ ━━━━━━━━━━━━━━  ☾

Step-by-step explanation:

-2-10h=-11-20, take -10 to the other side.

-2= -1-20, take 20 to the other side.

-18=-1h

-ve and -ve is pos.

h=18

positive 18

How to solve this proof?

How to solve this proof?

Answers

Answer:

∆ACD≅∆BCD                              definition of a segment bisector

Step-by-step explanation:

I think this is the only way to prove it in one reason

DE=DE (common)
AE=BE (CE intersects AB)
And angle AED = angle BED
So triangle ADE ≅ triangle BDE
(SAS congruence rule)

0.95 x 0.22 help pls??

Answers

The answer to the problem is 0.209

PLS HELP I GOTTA TURN THIS INNN ​

PLS HELP I GOTTA TURN THIS INNN

Answers

Answer:

Lauren

Step-by-step explanation:

1/2 is Erin

3/4 is Lauren

Answer:

1/2 for Eric and 6/8 for lauren

Eric: 1/2 = 4/8

Lauren: 6/8

Lauren was more succesful

Step-by-step explanation:

How many ways are there to arrange 12 (distinct) people in a row so that Dr. Tucker is 3 positions away from Dr. Stanley (i.e., 2 people are inbetween Dr. Tucker and Dr. Stanley), e.g., . . . . T _ _ S . . . .

Answers

There are 3,628,800 ways to arrange 12 distinct people in a row so that Dr. Tucker is 3 positions away from Dr. Stanley.

To count the number of arrangements of 12 people with Dr. Tucker and Dr. Stanley positioned 3 apart, we can treat Dr. Tucker and Dr. Stanley as a single block of two people, and then arrange the resulting 11 blocks in a row.

Since Dr. Tucker and Dr. Stanley can occupy any of the 10 possible positions (the first two positions, the second and third, and so on up to the last two positions), there are 10 ways to form this block.

After the block is formed, we are left with 10 remaining people to arrange in the remaining 10 positions. There are 10! ways to arrange these people, so the total number of arrangements is:

10 x 10! = 3,628,800

Therefore, there are 3,628,800 ways to arrange 12 distinct people in a row so that Dr. Tucker is 3 positions away from Dr. Stanley.

for such more question on word problem

https://brainly.com/question/13818690

#SPJ11

Find the solution to the system of equations: X – 2y + z = – 2 y + 2z = 5 x + y + 3z = 9 X = y = z =

Answers

The solution to the system of equations is (3, 1, 2).

We have to find the solution of given system of equations.

Given system of equations are:

X – 2y + z = – 2 y + 2z = 5 x + y + 3z = 9 X = y = z =

To find the value of x, we need to convert the given equations into standard form

.x - 2y + z = -2   ------------(1)

y + 2z = 5   ------------(2)

x + y + 3z = 9 ------------(3)

From equation (2), we get y = 5 - 2zy = (5 - 2z)

Putting this value of y in equation (1), we get:

x - 2(5 - 2z) + z = -2x - 10 + 4z + z = -2x + 5z = 8         ------------(4)

From equation (3), we get x + (5 - 2z) + 3z = 9x - 2z = 4          ------------(5)

Multiplying equation (4) by 2 and adding with equation (5), we get:2x + 10z = 16 + 4x - 8z2x - 12z = -16         ------------(6)

Adding equation (4) and equation (5),

we get:x - 2z + z = -2 + 4x - 2z + 4z = 8 + 4x2x + 2z = 10x + z = 5          ------------(7)

Adding equation (5) and equation (6), we get:4x - 10z = -204x - 5z = -10z = 2

Putting z = 2 in equation (7), we get:x + 2 = 5x = 3

Putting x = 3 and z = 2 in equation (2), we get:y + 2(2) = 5y = 1

The solution of given system of equations is x = 3, y = 1 and z = 2.

Hence, x = 3, y = 1 and z = 2.

To know more about system of equations click on below link:

https://brainly.com/question/20067450#

#SPJ11

Including a 6% sales tax, a new bike costs $514.1. Find the cost of the bike before
tax.

Answers

the cost of the bike before tax

514.1/106%=$485

Dave is considering two loans. Loan U has a nominal interest rate of 9. 97%, and Loan V has a nominal interest rate of 10. 16%. If Loan U is compounded daily and Loan V is compounded quarterly, which loan will have the lower effective interest rate, and how much lower will it be? a. Loan V’s effective rate will be 0. 3324 percentage points lower than Loan U’s. B. Loan V’s effective rate will be 0. 1187 percentage points lower than Loan U’s. C. Loan U’s effective rate will be 0. 5124 percentage points lower than Loan V’s. D. Loan U’s effective rate will be 0. 0713 percentage points lower than Loan V’s.

Answers

Loan U  will have a lower effective interest rate, and 0.0713 lower percentage points lower than Loan V and it can be determined by using the rate of interest formula.

Given that,

Dave is considering two loans.

Loan U has a nominal interest rate of 9.97%, and Loan V has a nominal interest rate of 10.16%.

If Loan U is compounded daily and Loan V is compounded quarterly.

We have to determine,

Which loan will have the lower effective interest rate, and how much lower will it be?

According to the question,

Effective Interest Rate;

The effective interest rate is determined by the formula;

\(= \left(1+\dfrac{r}{t} \right )^t\)

Loan U has a nominal interest rate of 9.97%,

Loan U is compounded daily,

Then,

The effective annual multiplier for loan U is,

\(= \left(1+\dfrac{0.997}{365}\right)^{365}\\\\= (1+0.0027)^{365}\\\\= (1.0027)^{365}\\\\= 1.104824\)

And Loan V has a nominal interest rate of 10.16%,

and Loan V is compounded quarterly.

Then,

\(= \left(1+\dfrac{0.1016}{4}\right)^{4}\\\\= (1+0.0254)^{365}\\\\= (1.0254)^{365}\\\\= 1.105537\)

Therefore,

Loan V has a higher effective rate by,

\(=1.105537 -1.104824 \\\\= 0.000713 \\\\= 0.0713%\)

Hence, Loan U  will have a lower effective interest rate, and 0.0713 lower percentage points lower than Loan V.

For more details about Interest rate refer to the link given below.

https://brainly.com/question/1398822

 

What does multiplicity mean in math?

Answers

The phrase "number of values for which a certain condition holds" is referred to as multiplicity. The phrase, for instance, can be used to describe the magnitude of the totient valence function or the frequency with which a given polynomial equation has a root at a specific location.

Let z_0 be a root of a function f, and let n be the least positive integer n such that f^((n))(z_0)!=0. Then the power series of f about z_0 begins with the nth term,

f(z)=sum_(j=n)^infty1/(j!)(partial^jf)/(partialz^j)|_(z=z_0)(z-z_0)^j,

and f is said to have a root of multiplicity (or "order") n. If n=1, the root is called a simple root '

The multiplicity of a member of a multiset in mathematics is the number of times the member appears in the multiset. The multiplicity of a root, for instance, is how many times a given polynomial has a root at a particular point.

It's crucial to understand the concept of multiplicity in order to correctly count without mentioning exceptions (for example, double roots counted twice). Thus, "counted with multiplicity" is used.

This can be highlighted by counting the number of different elements, as in "the number of separate roots," if multiplicity is disregarded. However, multiplicity is always taken into account when a set (as opposed to a multiset) is established, therefore the word "different" is not necessary.

Hence ,Multiplicity means the quality or state of being multiple or various. and

the number of components in a system (such as a multiplet or a group of energy levels)

Tp know more about Mutiplicity visit : brainly.com/question/5992872

#SPJ4

Let z_0 be a root of a function f, and let n be the least positive integer n such that f^((n))(z_0)!=0. Then the power series of f about z_0 begins with the nth term,

f(z)=sum_(j=n)^infty1/(j!)(partial^jf)/(partialz^j)|_(z=z_0)(z-z_0)^j,

and f is said to have a root of multiplicity (or "order") n. If n=1, the root is called a simple root '

The multiplicity of a member of a multiset in mathematics is the number of times the member appears in the multiset. The multiplicity of a root, for instance, is how many times a given polynomial has a root at a particular point.

The point (8,2) is dilated using a scale factor of 2. Where is the image point located?

Answers

The image point is located at (16, 4).

What is Dilation?

Dilation is a type of transformation where the figure is enlarged or made smaller such that it preserves the shape but not size.

Every dilated image are similar figures to the original figure.

Given that, a point (8, 2) is dilated using a scale factor of 2.

A point (x, y) when dilated by a scale factor of k changes to (kx, ky).

Point (8, 2) will become,

(8 × 2, 2 × 2) = (16, 4).

Hence the dilated point is (16, 4).

To learn more about Dilation, click on the link :

https://brainly.com/question/13176891

#SPJ1

Find the domain of the vector-valued function. r(t) = f(t) × g(t), where f(t) = t3i − tj tk, g(t) = 3 t i 1 t 2 j (t 8)k

Answers

The given vector-valued function is\(r(t) = f(t) × g(t)\), where \(f(t) = t^3i − tj − tk\) and\(g(t) = 3ti + t^2j − (t + 8)k.\)

To find the domain of the vector-valued function, we need to determine the values of t for which both f(t) and g(t) are defined.

For f(t), there are no restrictions on the domain since it is defined for all real values of t.

For g(t), we need to consider the denominator (t + 8) in the k-component. To avoid division by zero, we set t + 8 ≠ 0 and solve for t: t ≠ -8.

Therefore, the domain of the vector-valued function r(t) is all real numbers except t = -8.

Main answer: The domain of the vector-valued function

\(r(t) = f(t) × g(t) is (-∞, -8) U (-8, ∞).\)

The domain of the vector-valued function \(r(t) = f(t) × g(t)\)can be found by determining the values of t for which both f(t) and g(t) are defined. The function\(f(t) = t^3i − tj − tk\)is defined for all real values of t since there are no restrictions on its domain.

However, for the function \(g(t) = 3ti + t^2j − (t + 8)k\),

we need to consider the denominator (t + 8) in the k-component. To avoid division by zero, we set t + 8 ≠ 0 and solve for t: t ≠ -8. Therefore, the domain of g(t) is all real numbers except t = -8. Finally, to find the domain of r(t), we need to consider the intersection of the domains of f(t) and g(t), which is (-∞, -8) U (-8, ∞).

The domain of the vector-valued function \(r(t) = f(t) × g(t) is (-∞, -8) U (-8, ∞).\)

To know more about real numbers  :

brainly.com/question/9876116

#SPJ11

Malachy, Sushil and Fiona share some sweets in the ratio 1:3:1. Malachy gets 5 sweets. How many did Sushil get?

Answers

Answer:

Sushil got 15 sweets

Step-by-step explanation:

we can use law of indices property as shown below

a:b:c = ax:bx:cx

that if we multiply each term of ratio by same number , the ratio does not change

____________________________

given

Malachy, Sushil and Fiona share some sweets in the ratio 1:3:1.

let

Malachy gets 1x sweets

sushil gets 3x sweets

Fiona gets 1x sweets

but given that

Malachy got 5 sweets

thus

1x = 5

x = 5

Sushil got 3x sweets = 3*5 = 15 sweets.

Sushil got 15 sweets.

how do you do: -x-x-3x+2-5 ?

Answers

Answer:

=−5x−3

Step-by-step explanation:

Let's simplify step-by-step.

−x−x−3x+2−5

=−x+−x+−3x+2+−5

Combine Like Terms:

=−x+−x+−3x+2+−5

=(−x+−x+−3x)+(2+−5)

=−5x+−3

brainliest please?

What is the length of the hypotenuse? If necessary, round to the nearest tenth.
C=
yards
4 yd.
3yd.

Answers

Answer:

5yds

Step-by-step explanation:

\(a^{2} +b^{2} =c^{2}\)

\(4^{2} +3^{2} =c^{2}\)

16+9=\(c^{2}\)

25 = \(c^{2}\)

\(\sqrt{25\) = c

5= c

5 yds

Other Questions
List out four-four natural and man-made hazards. Write an application to your coordinator for your educational tour In what country do cherry blossoms signify spring? ATC 14-1 Business Applications Case Preparing and using pro forma statementsMary Helu and Jason Haynes recently graduated from the same university. Aftergraduation they decided not to seek jobs at established organizations but, rather, to starttheir own small business hoping they could have more flexibility in their personal livesfor a few years. Marys family has operated Mexican restaurants and taco trucks for thepast two generations, and Mary noticed there were no taco truck services in the townwhere their university was located. To reduce the amount they would need for an initialinvestment, they decided to start a business operating a taco cart rather than a tacotruck, from which they would cook and serve traditional Mexican-styled street food.They bought a used taco cart for $18,000. This cost, along with the cost for supplies toget started, a business license, and street vendor license brought their initialexpenditures to $22,000. They took $5,000 from personal savings they hadaccumulated by working part time during college, and they borrowed $17,000 fromMarys parents. They agreed to pay interest on the outstanding loan balance each monthbased on an annual rate of 5 percent. They will repay the principal over the next fewyears as cash becomes available. They were able to rent space in a parking lot near thecampus they had attended, believing that the students would welcome their food as analternative to the typical fast food that was currently available.After two months in business, September and October, they had average monthlyrevenues of $25,000 and out-of-pocket costs of $22,000 for rent, ingredients, papersupplies, and so on, but not interest. Jason thinks they should repay some of the moneythey borrowed, but Mary thinks they should prepare a set of forecasted financialstatements for their first year in business before deciding whether or not to repay anyprincipal on the loan. She remembers a bit about budgeting from a survey of accountingcourse she took and thinks the results from their first two months in business can beextended over the next 10 months to prepare the budget they need. They estimate thecart will last at least four years, after which they expect to sell it for $6,000 and move onto something else in their lives. Mary agrees to prepare a forecasted (pro forma) incomestatement, balance sheet, and statement of cash flows for their first year in business,which includes the two months already passed.Requireda. Prepare the annual pro forma financial statements that you would expect Mary toprepare based on her comments about her expectations for the business. Assume noprincipal will be repaid on the loan. [Note: Some amounts may be differentfrom the printed text version]b. Review the statements you prepared for the first requirement and prepare a list ofreasons why actual results for Jason and Marys business probably will not matchtheir budgeted statements.Please show how you got the answers to the INCOME STATEMENT, BALANCE SHEET, AND STATEMENT OF CASH FLOWS.Thank You! How did La Raza use resistance to live up to our idea of freedom, equality, and democracy? eory.According to cell theory, which of the following aremade of cells? Check all that apply.O flowersrocksbloodwaterbacteriasugarskinRETRY Which of the following is true concerning the fluid compartmentsof the body?A. Total body water and the fluid volumes of each compartmentchange very quite a bit from day to dayB. Movement of water Explain how to translate the phrase into an algebraicexpression.four times the difference of eight and a numher What is the solution to the division problem below? (You can use long division or synthetic division) X^3 + x^2 - 11x + 4 / x + 4A.x^2-4x+1B.x^2-3x+1C.x^2-6x+1D.x^2-5x+1 What is the similarities between negative feedback and positive feedback Write a C# program named DoubleDecimalTest thatdeclares and displays two variables - a double and a decimal.Experiment by assigning the same constant value to each variable sothat the assignment to the double is legal but the assignment tothe decimal is not. In other words, when you leave the decimalassignment statement in the program, an error message should begenerated that indicates the value is outside the range of the typedecimal, but when you comment out the decimal assignment and itsoutput statement, the program should compile correctly. What challenges does kfc face in the micro environment 1800 people attended the National Day dinner in a certain constituency Given that 35.5 of them were. men, 40 % of them were women and the rest were children, find the number of children who attended the dinner please answer!!! Express cos J as a fraction in simplest terms. select the correct statement. a.) the critical z-score for a right-tailed test at a 9% significance level is 1.34. b.) the critical z-score for a two-sided test at a 4% significance level is 1.75. c.) the critical z-score for a two-sided test at a 20% significance level is 0.85. d.) the critical z-score for a left-tailed test at a 12% significance level is -0.45. If the price of good x becomes lower, then the level of consumer surplus becomes? What is a 3 word phrase with the letters airthutnlusrhngama that tells what the bill of rights protects? Aquick3. Prompt: Which quality is more important in a leader:decision-making or strong public speakingskills? Support your response with evidence from thenovel as well as from your own personal experienceand observations.Consider the prompt and choose a side to argue.Write a thesis that includes a counterclaim.You may use the sentence frames tab to help youwrite your thesis with a counterclaim. voter behavior that evaluates candidates based on forecasts of their future political behavior is known as what troubleshooting steps should you take if a shared printer is printing out control codes on output pages?